Q11

User avatar
 
LSAT-Chang
Thanks Received: 38
Atticus Finch
Atticus Finch
 
Posts: 479
Joined: June 03rd, 2011
 
 
trophy
Most Thankful
trophy
First Responder
 

Q11

by LSAT-Chang Tue Sep 20, 2011 12:54 am

I picked B at first but I guess passage B wont agree with this. For evidence for A, would lines 11-20 suffice? And from passage B would it be lines 53-55 be the correct line reference? Please correct me if there is evidence elsewhere in the passage...
User avatar
 
maryadkins
Thanks Received: 641
Atticus Finch
Atticus Finch
 
Posts: 1261
Joined: March 23rd, 2011
 
This post thanked 1 time.
 
 

Re: Q11

by maryadkins Thu Sep 22, 2011 1:57 pm

Yes--those are sufficient lines to support (A).

(B) is incorrect because the author of Passage B would not agree with this.

(C) is incorrect. The author of Passage A says the opposite (line 21).

(D) contradicts Passage B.

(E) Passage B says nothing about this.
 
Lsat 123
Thanks Received: 0
Vinny Gambini
Vinny Gambini
 
Posts: 4
Joined: July 30th, 2014
 
 
 

Re: Q11

by Lsat 123 Thu Sep 04, 2014 8:29 pm

thanx
Last edited by Lsat 123 on Sun May 31, 2015 10:11 pm, edited 1 time in total.
User avatar
 
tommywallach
Thanks Received: 468
Atticus Finch
Atticus Finch
 
Posts: 1041
Joined: August 11th, 2009
 
 
 

Re: Q11

by tommywallach Mon Sep 08, 2014 9:09 pm

The answer is (A).

-t
Tommy Wallach
Manhattan LSAT Instructor
twallach@manhattanprep.com
Image
 
tcobbin
Thanks Received: 2
Vinny Gambini
Vinny Gambini
 
Posts: 2
Joined: June 07th, 2014
 
 
 

Re: Q11

by tcobbin Wed Dec 10, 2014 12:49 pm

Why is C incorrect, even though the author states "Glyphosate has been successfully used," the author still states that some form of "integrated control" is still needed to control the population, which suggests that the control method is not completely effective? Thank you.
User avatar
 
maryadkins
Thanks Received: 641
Atticus Finch
Atticus Finch
 
Posts: 1261
Joined: March 23rd, 2011
 
This post thanked 1 time.
 
 

Re: Q11

by maryadkins Fri Dec 12, 2014 9:45 am

Saying integrated control is needed is different from saying that something is "very difficult" to control.

I could say, "I need an umbrella today," but that doesn't mean I think it's "very difficult" to find one or to deal with rain. If anything, the author of Passage A seems to think control is quite possible, it just hasn't happened yet.

Hope this helps!
 
giuffre2180
Thanks Received: 0
Vinny Gambini
Vinny Gambini
 
Posts: 1
Joined: July 29th, 2015
 
 
 

Re: Q11

by giuffre2180 Wed Jul 29, 2015 9:34 pm

Looks like no one has replied to this section/question in a while so I might not get a response for sometime. But this question has been bugging me, as I choose B, and cannot figure out why it is incorrect. We are looking for answer the two passages would most likely agree with and Passage A would obviously agree with B, but the last line in Passage B I feel also agrees with choice B. Please clarify if I am missing something :?
 
bluebeans
Thanks Received: 0
Vinny Gambini
Vinny Gambini
 
Posts: 2
Joined: August 23rd, 2015
 
 
 

Re: Q11

by bluebeans Sun Sep 06, 2015 3:23 pm

I also felt inclined to (B) due to the mention of "wetland" in the last line of Passage B.

But if you look at how PsgB mentions the effects of purple loosestrife on wetland vegetation, you notice that he doesn't think it's that big of a deal. PsgB says the real motivation for everyone flipping out is the "economics of exploiting", which is now under threat b/c of a "decline in the production of the wetland resource." In PsgB, the decline of wetland is mentioned matter-of-factly. Sure, there's a decline, but PsgB doesn't really think its a big deal -- instead it mentions the decline only to explain its relationship to the "economics of exploiting".

If answer choice (B) read, "Its establishment.... has had an effect" rather than "Its establishment... has had a disastrous effect", then it would've been fine. But does PsgB really think the effect on wetland has been disastrous? Naah.